15 svar
680 visningar
mrlill_ludde 1047 – Fd. Medlem
Postad: 10 aug 2019 15:59 Redigerad: 10 aug 2019 16:03

Eulers fi funktion, en gång för alla!

I denna tråd https://www.pluggakuten.se/trad/euler-fi-funktion/ så sa man att man kan utnyttja fi(2m) = 2*fi(m) . Men inte för fi(3m) = 3*fi(m) Men i den här https://www.pluggakuten.se/trad/order-i-grupper-men-u-z_m/ skriva man att det inte är sant. 


Enl den här viden https://www.khanacademy.org/computing/computer-science/cryptography/modern-crypt/v/euler-s-totient-function-phi-function så skriver en man:

 

"That is a consequence of the fact that the totient function is multiplicative. If p is a sufficiently large prime, for instance,
φ(2p) = φ(2) + φ(p) = 1 + (p-1) = p
φ(3p) = φ(3) + φ(p) = 2 + (p-1) = p + 1
φ(5p) = φ(5) + φ(p) = 4 + (p-1) = p + 3
So those lines that you see have slopes of 1/2, 1/3, 1/5, 1/7, 1/11, and so on."

 


Så jag måste veta.. haha. Hur står det till egentligen?

 

Vi tar ett exempel. fi(100) .

det kan man ju göra på kanske dessa alternativ:

  1. fi(100) = 2*fi(50) = 2*fi(25) sen räkna alla som är koprima (eller heter det så?) alltså 2*20 = 40
  2. φ(5p) = φ(5) + φ(p) = 4 + (p-1) = p + 3 blir då för mig: p=20 (eftersom 5*20=100), alltså 23? eller?
Smaragdalena Online 78141 – Lärare
Postad: 10 aug 2019 18:00

2. φ(5p) = φ(5) + φ(p) = 4 + (p-1) = p + 3 blir då för mig: p=20 (eftersom 5*20=100), alltså 23? eller?

Vad har du för värde på p här?

Micimacko 4070
Postad: 10 aug 2019 18:09 Redigerad: 10 aug 2019 18:16

Är inte helt säker på vad du frågar, men jag räknar ut det såhär. Hoppas du ser mönstret, fick inte ihop det i ord 😅. Tänk på att en multiplikativ funktion bara behöver vara det för tal som saknar genemsamma primfaktorer, annars kan lite vad som helst hända.

mrlill_ludde 1047 – Fd. Medlem
Postad: 10 aug 2019 19:00
Smaragdalena skrev:

2. φ(5p) = φ(5) + φ(p) = 4 + (p-1) = p + 3 blir då för mig: p=20 (eftersom 5*20=100), alltså 23? eller?

Vad har du för värde på p här?

jag antar p=20, eftersom 5p=100?

Micimacko 4070
Postad: 10 aug 2019 19:11

Står inte p för primtal? Isf är 20 ett dåligt val.

Laguna Online 28435
Postad: 10 aug 2019 19:12

Du skulle kunna titta på den engelska eller svenska wikipedia-sidan också. 

Smaragdalena Online 78141 – Lärare
Postad: 10 aug 2019 19:12 Redigerad: 10 aug 2019 19:22
mrlill_ludde skrev:
Smaragdalena skrev:

2. φ(5p) = φ(5) + φ(p) = 4 + (p-1) = p + 3 blir då för mig: p=20 (eftersom 5*20=100), alltså 23? eller?

Vad har du för värde på p här?

jag antar p=20, eftersom 5p=100?

Det står i förutsättningarna i satsen att OM p är ett tillräckligt stort primtal, så... Är 20 ett primtal? Kan man alltså använda satsen så som du har gjort?

Albiki 5096 – Fd. Medlem
Postad: 10 aug 2019 20:10 Redigerad: 10 aug 2019 20:10

Om nn är ett positivt heltal så betecknar ϕ(n)\phi(n) antalet positiva heltal (mindre än nn) som är relativt prima med nn; exempelvis är ϕ(10)=4\phi(10) = 4 eftersom talen 1, 3, 7, 9 saknar gemensam delare med 10 (är relativt prima med 10).

Om pp är ett primtal så är alla positiva heltal (mindre än pp) relativt prima med pp, så för ett primtal är ϕ(p)=p-1\phi(p) = p-1.

Albiki 5096 – Fd. Medlem
Postad: 10 aug 2019 20:15 Redigerad: 10 aug 2019 20:18

För varje positivt heltal nn gäller det att

    k·nloglogn<ϕ(n)n-1k\cdot \frac{n}{\log\log n} < \phi(n) \leq n-1

och det finns heltal där den övre gränsen antas (primtalen!) men det finns inte heltal där den nedre gränsen antas (eftersom den nedre gränsen är aldrig ett heltal), där kk är en viss positiv konstant.

mrlill_ludde 1047 – Fd. Medlem
Postad: 10 aug 2019 20:17 Redigerad: 10 aug 2019 20:19
Micimacko skrev:

Står inte p för primtal? Isf är 20 ett dåligt val.

Nvm

Albiki 5096 – Fd. Medlem
Postad: 10 aug 2019 20:23 Redigerad: 10 aug 2019 20:28

Om mm och nn är två positiva heltal så gäller det att

    ϕ(m·n)=ϕ(m)·ϕ(n)·dϕ(d)\phi(m\cdot n)=\phi(m)\cdot\phi(n)\cdot\frac{d}{\phi(d)}

där dd betecknar den största gemensamma delaren till mm och nn; om mm och nn är relativt prima är d=1d = 1 och ϕ(1)=1\phi(1) = 1 varför funktionen ϕ\phi är multiplikativ då den appliceras på heltal som är relativt prima.

Exempelvis är ϕ(2·5)=ϕ(2)·ϕ(5)·dϕ(d)\phi(2\cdot 5) = \phi(2)\cdot \phi(5) \cdot \frac{d}{\phi(d)} men talen 22 och 55 är primtal och saknar därför gemensam delare, utom talet 11 förstås vilket då blir den största gemensamma delaren till 22 och 55, vilket ger ϕ(2·5)=ϕ(2)·ϕ(5).\phi(2\cdot 5) = \phi(2)\cdot \phi(5).22 och 55 är primtal gäller det att ϕ(2)=2-1\phi(2) = 2-1 och ϕ(5)=5-1\phi(5) = 5-1 vilket ger resultatet ϕ(10)=4.\phi(10) = 4.

Albiki 5096 – Fd. Medlem
Postad: 10 aug 2019 20:37

Det är känt att mängden MM är en tät delmängd av det öppna intervallet (0,1)(0,1). Vad säger detta om funktionen ϕ\phi?

    M={ϕ(n)n:n=1,2,3,}.M = \{\frac{\phi(n)}{n}\,:\, n = 1,2,3,\ldots\}.

Albiki 5096 – Fd. Medlem
Postad: 10 aug 2019 20:41

Det beräknades att ϕ(10)=4\phi(10) = 4. Finns det något annat positivt heltal med samma ϕ\phi-värde som 1010

mrlill_ludde 1047 – Fd. Medlem
Postad: 12 aug 2019 16:28
Micimacko skrev:

Är inte helt säker på vad du frågar, men jag räknar ut det såhär. Hoppas du ser mönstret, fick inte ihop det i ord 😅. Tänk på att en multiplikativ funktion bara behöver vara det för tal som saknar genemsamma primfaktorer, annars kan lite vad som helst hända.

Så man primtalsfaktoriserar?

den e jag inte med på, vad e det för formel?

mrlill_ludde 1047 – Fd. Medlem
Postad: 12 aug 2019 16:34
Albiki skrev:

Det beräknades att ϕ(10)=4\phi(10) = 4. Finns det något annat positivt heltal med samma ϕ\phi-värde som 1010

kan det va alla multiplar?

Albiki 5096 – Fd. Medlem
Postad: 12 aug 2019 16:37 Redigerad: 12 aug 2019 16:39
mrlill_ludde skrev:
Albiki skrev:

Det beräknades att ϕ(10)=4\phi(10) = 4. Finns det något annat positivt heltal med samma ϕ\phi-värde som 1010

kan det va alla multiplar?

Undersök själv. Är ϕ(10·2)=4\phi(10\cdot 2) = 4? För beräkningen kan du utnyttja 10·2=4·510\cdot 2=4\cdot 5; anledningen är att 44 och 55 är relativt prima, men 22 och 1010 är det inte.

Svara Avbryt
Close